1959 AHSME Problems/Problem 7

Revision as of 05:53, 24 September 2018 by Dajeff (talk | contribs) (Created page with "==Solution== If we let <math>a=3</math> and <math>d=1</math>, then we will get a <math>3</math>-<math>4</math>-<math>5</math> triangle, which is a right triangle. So, the answ...")
(diff) ← Older revision | Latest revision (diff) | Newer revision → (diff)

Solution

If we let $a=3$ and $d=1$, then we will get a $3$-$4$-$5$ triangle, which is a right triangle. So, the answer is $\boxed{\textbf{D}}$.